Recent content by giodude

  1. giodude

    How Do You Calculate Wire Extension Under Sudden Loads?

    Oh, think I figured it out. This might be where the inexactitude you mentioned is coming in: The ##x## in the previous message is the amplitude. Now we can plug it into ##mgh = \frac{1}{2}kA^{2}## which we derive from the conservation of energy equation: ##h = \frac{kA^{2}}{2mg} = 0.2536m##...
  2. giodude

    How Do You Calculate Wire Extension Under Sudden Loads?

    We also know ##T = 216 N## from the original post, given the ultimate strength of the wire. So, ##T = kx - mg## ##216 N = kx - (0.5 kg)(9.8 \frac{m}{s^{2}})## ##kx = 220.9 N## ##x = \frac{220.9 N}{k} = 0.0113 m## Dimensionally this resolves properly. However, the solution in the textbook is...
  3. giodude

    How Do You Calculate Wire Extension Under Sudden Loads?

    Whats the inexactitude? Also how does this reconcile with ##v_{f} = \sqrt{2gh}## if ##v = 0## at time ##t##?
  4. giodude

    How Do You Calculate Wire Extension Under Sudden Loads?

    Oh, I see. If ##T = kx - mg = ma## is at its maximum then acceleration needs to be maximum. As a result, we want ##cos(\omega t + \alpha) = 1## and ##sin(\omega t + \alpha) = 0##. This makes sense because this sets ##x = Acos(\omega t + \alpha)## to its max ##x = A##.
  5. giodude

    How Do You Calculate Wire Extension Under Sudden Loads?

    We also know that in SHM ##x = A cos(\omega t + \alpha)##, where ##\omega^{2} = \frac{k}{m}##. So, ##v = \frac{dx}{dt} = - \omega A sin(\omega t + \alpha)## ##a = \frac{d^{2}x}{dt^{2}} = - \omega^{2} A cos(\omega t + \alpha)## KE is maximum where ##a = 0## at ##cos(\omega t + \alpha) = 0## and...
  6. giodude

    How Do You Calculate Wire Extension Under Sudden Loads?

    I think I'm starting to get somewhere. We know, due to the starting state of lifting and dropping that the initial speed, ##v_{0} = 0##. Additionally, we know that ##KE = mgh## at the point of peak tension. Therefore, ## \Delta KE = \frac{1}{2} \times m \times v_{f}^{2} - \frac{1}{2} \times m...
  7. giodude

    How Do You Calculate Wire Extension Under Sudden Loads?

    It'd occur just before the sudden jerk? So, at PE = 0 and KE = mgh?
  8. giodude

    How Do You Calculate Wire Extension Under Sudden Loads?

    The change in GPE will be mgh from start to finish. Which means KE will have changed from 0 to mgh at the moment of maximum tension.
  9. giodude

    How Do You Calculate Wire Extension Under Sudden Loads?

    Hi, I solved part (a) and will provide my solution below. However, I've been working on part (b) for quite a bit and reviewed the provided, relevant text a few times now but haven't been able to find what I'm missing: Solution (a): Using ##A = \frac{\pi \times d^{2}}{4}##, ##k =...
  10. giodude

    I Superconductivity energy saved v Cooling/Heating energy loss

    Hi! In reading about Superconductivity and its current state of only being achieved in super cooled or heated materials. This sparked a question the following question: What is the result of the trade off between energy saved by avoiding dissipation through the natural resistance of a material...
  11. giodude

    I Electricity and Magnetism: Verifying the Inverse Square Law

    I took images of them on my phone and then airdropped them to my laptop, I'll fix them up when I get back to my laptop and edit the post. Thank you.
  12. giodude

    I Electricity and Magnetism: Verifying the Inverse Square Law

    Hello, I'm currently working through Purcell and Morin, Electricity and Magnetism textbook and came across a problem in which the goal is to verify the inverse square law. I'm worked through and completed the problem. However, I'm confused how this verifies the inverse square law, I'm posting...
  13. giodude

    MIT OCW 8.01 PS11.3: Elastic Collision Between Ball and Pivoted Rod

    Ah, I think my sign error is a result of improperly setting up my cross product calculation with respect to the final velocity. Once fixed it looks like the sign is fixed because what was previously ##L_{P, f}^{sys} = I_{cm} + md^{2} + \frac{1}{2}dmv_{f}## becomes ##L_{P, f}^{sys} = I_{cm} +...
  14. giodude

    MIT OCW 8.01 PS11.3: Elastic Collision Between Ball and Pivoted Rod

    Fixed it using conservation of angular momentum: System: ##L_{P,i}^{sys} = L_{P,f}^{sys}## ##k_{i} = k_{f}## Solve for initial and final angular momentum about ##P##: ##L_{P,i}^{sys} = -\frac{d}{2}mv_{i}## ##L_{P,f}^{sys} = I_{cm} + md^{2} + \frac{d}{2}mv_{f} = \frac{1}{3}md^{2}\omega_{f} +...
Back
Top